Chang knows one side of a triangle is 13 cm. Which set of two sides is possible for the lengths of the other two
sides of this triangle?
5 cm and 8 cm
6 cm and 7 cm
7 cm and 2 cm
8 cm and 9 cm

Answers

Answer 1

Answer:

8 cm and 9 cm

Step-by-step explanation:

Hi there!

The sum of the lengths of two sides of a triangle must always be greater than the length of the third side.

5 cm and 8 cm ⇒ 5+8=13; not greater than 13

6 cm and 7 cm ⇒ 6+7=13; not greater than 13

7 cm and 2 cm ⇒ 7+2=9; not greater than 13

8 cm and 9 cm ⇒ 8+9=17; greater than 13

Therefore, the last set of two sides is possible for the lengths of the the other two sides of this triangle.

I hope this helps!

Answer 2
8cm and 9 cm because the sum of the two sides must be greater than 13.

5+8=13
6+7=13
7+2=9
8+9=17

Related Questions

Will mark BRAINLIEST for correct answer.

The music you hear when listening to a magnetic cassette tape is the result of the magnetic tape passing over magnetic heads, which read the magnetic information on the tape. The length of time a tape will play depends on the length of the tape and the operating speed of the tape player. The formula is T = L/S, where T is the time in seconds, L is the length of the tape in inches, and S is the operating speed in inches per second.

How long a tape does a police officer need to record a 3-minute confession at an operating speed of 3 1/2 inches per second?

Answers

Answer:

L = 3.5 in/sec  * 3 min  * 60 sec /min = 630 in

(L = S * T   where T is in sec and S is inches / sec)

I need help with my math!!!!!!!

Answers

Answer:

2[tex]\sqrt{6}[/tex]

Step-by-step explanation:

Area of a square = [tex]s^{2}[/tex]   (s is a side length)

24 = [tex]s^{2}[/tex]

[tex]\sqrt{24}[/tex] = [tex]\sqrt{s^{2} }[/tex]

[tex]\sqrt{24}[/tex] = s

[tex]\sqrt{(4)(6) }[/tex] = s

2[tex]\sqrt{6}[/tex] = s

3. The following are the scores you shot in five rounds of golf: 70,80,75,75,80. The variance is: A. 3.74 C. 14 B. 196 D. 70​

Answers

Answer:

Option C.

Step-by-step explanation:

For a given set of N elements:

{x₁, x₂, ..., xₙ}

The mean is given by:

[tex]M = \frac{x_1 + x_2 + ... + x_n}{N}[/tex]

And the variance is given by:

[tex]v = \frac{(x_1 - M)^2 + (x_2 - M)^2 + ... + (x_n - M)^2}{N}[/tex]

Then for our set:

{70, 80 ,75, 75, 80}

We have 5 elements, then N = 5

The first thing we need to find is the mean.

[tex]M = \frac{70 + 80 + 75 + 75 + 80}{5} = 76[/tex]

Then the variance will be:

[tex]v = \frac{(70 - 76)^2 + (80 - 76)^2 + (75 - 76)^2 + (75 - 76)^2 + (80 - 76)^2}{5} = 14[/tex]

The correct option is C, 14.

If 260 people donated $10.00 to an organization how much money would said organization have received in total?

Answers

Answer:

$ 2,600

Step-by-step explanation:

It's given that 260 People donated $10 is to an organisation . We need to find the total money received by the organisation .

Here we just need to the number of people with the amount each person gave , that is ;

[tex]\implies \rm Total \ amount = \$ 10 \times 260 \\\\\rm\implies Total \ amount = \$ 2,600 [/tex]

Hence the total money received by the organisation is $ 2600 .

Solve for x. Round to the nearest tenth of a degree, if necessary

Answers

Step-by-step explanation:

here's the answer to your question

Can someone help me solving this problem? I'm confuse so I want help..
question :
a) The length and breadth of a rectangular room are 10m and 8m respectively. How many pieces of carpet 5m long and 2m wide are required to cover it's floor?
Please help me solving this problem !

Answers

Answer: 8

Step-by-step explanation:

Write an equation in point-slope form of the line through point J(-1, 10) with slope -7.

Answers

Answer:

y - 10 = -7 (x + 1)

Step-by-step explanation:

pont = (-1,10)     slope = -7

y - 10 = -7 (x + 1)

The price of a jacket usually costing £50 is increased to £65.
Work ou the percentage increase.

Answers

Answer:

So the answer is 30%

Step-by-step explanation:

1. For 50 = 15 increase euros

2. For 100 = 15/50 x 100 = 30 increase in euros

3. 30/100 = 30%

OR....

1. Ten percent of 50 is 5.

2. 5x3= 15(increase in euros)

3. 10 percent three times which is equal to 30%.

A bottling company marks a 0 for every bottle that comes out correct and a 1 for every defective bottle. Estimate the probability that the next bottle is defective

Answers

Answer:

See Explanation

Step-by-step explanation:

Given

[tex]0 \to[/tex] Correct

[tex]1 \to[/tex] Defective

Required

The probability that the next is defective

The question is incomplete because the list of bottles that came out is not given.

However, the formula to use is:

[tex]Pr = Num ber\ o f\ d e f e c t i v e \div T o t a l\ b o t t l e s[/tex]

Take for instance, the following outcomes:

[tex]0\ 1\ 0\ 0\ 0\ 1\ 0\ 0\ 1\ 1\ 0\ 0\ 0\ 1[/tex]

We have:

[tex]Total = 14[/tex]

[tex]D e f e ctive = 9[/tex] --- i.e. the number of 0's

So, the probability is:

[tex]Pr = 9 \div 14[/tex]

[tex]Pr = 0.643[/tex]

Answer:

1/20

Step-by-step explanation:

000000000000100000

It is asking the probability of a defective bottle

there is one defective bottle out of 20

How can you represent 1/2 on a 10-by-10 grid?

Answers

Answer:

Represent 1/2 by covering 50 squares.

Step-by-step explanation:

There are 100 squares in a 10-by-10 grid.

1/2 of 100 is 50, so you should cover 50 squares out of 100 squares.

what is the measure of 2?

Answers

Answer:

Value of x:

[tex]{ \tt{(7x + 1) \degree + (18x + 4) \degree = 180 \degree}} \\ { \tt{25x + 5 = 180}} \\ { \tt{25x = 175}} \\ x = 7[/tex]

Finding m‹2 :

[tex]{ \tt{m \angle2 = (7x + 1) \degree}} \\ { \tt{m \angle2 = (7 \times 7) + 1}} \\ { \bf{m \angle2 = 50 \degree}}[/tex]

Answer:

m∠2 = 50

Step-by-step explanation:

7x + 1 and 18x + 4 are angles in a linear pair.

Sum of linear pair angles is supplementary.

7x + 1 + 18x + 4 = 180

7x + 18x + 1 + 4 = 180

25x + 5 = 180

25x = 180 - 5

25x = 175

x = 175 / 25

x = 7

Substitute x = 7 in 7x + 1,

7x + 1

= 7 ( 7 ) + 1

= 49 + 1

= 50

7x + 1 and ∠2 are vertically opposite angles and vertically opposite angles are equal.

∠2 = 7x + 1

∠2 = 50

If 12x = 60, what is the last step to solve for x?

A. multiply 12 to both sides

B. subtract 12 from both sides

C. add 12 to both sides

D. divide 12 from both sides

Answers

Answer:

D. divide 12 from both sides

Step-by-step explanation:

12x = 60,

Divide each side by 12

12x/12 = 60/12

x = 5

We have to find,

The last step to solve for x,

→ 12x = 60

Then use option (D),

D. divide 12 from the both sides,

→ 12x = 60

→ 12x/12 = 60/12

→ x = 5

Hence, option (D) is last step.

Three different non-zero digits can be arranged in six different ways to
form six three-digit numbers. If the smallest three of these numbers add
to 540, what is the sum of the largest three numbers?

Answers

Answer:

1134

Step-by-step explanation:

We have 3 digits:

a, b, c

a 3 digit number can be written as:

a*100 + b*10 + c*1

Such that these numbers can be:

{1, 2, 3, 4, 5, 6, 7, 8, 9}

Let's assume that:

a < b < c

Then the 3 smaller numbers are:

a*100 + b*10 + c

a*100 + c*10 + b

b*100 + a*10 + c

The 3 larger numbers are:

b*100 + c*10 + a

c*100 + a*10 + b

c*100 + b*10 + a

We know that the sum of the 3 smaller numbers is equal to 540, then:

(a*100 + b*10 + c) + (a*100 + c*10 + b) + (b*100 + a*10 + c) = 540

Let's simplify this:

(a + a + b)*100 + (b + c + a)*10 + (c + b + c) = 540

(2a + b)*100 + (b + c + a)*10 + (2c + b) = 540

The sum of the 3 larger numbers is equal to X, we want to find the value of X:

(b*100 + c*10 + a) + (c*100 + a*10 + b) + (c*100 + b*10 + a) = X

Now let's simplify the left side:

(b + c + c)*100 + (c + a + b)*10 + (a + b + a)*1 = X

(b + 2*c)*100 + (c + a + b)*10 + (2a + b) = X

Then we have two equations:

(2a + b)*100 + (b + c + a)*10 + (2c + b) = 540

(b + 2*c)*100 + (c + a + b)*10 + (2a + b) = X

Notice that the terms are inverted.

By looking at the first equation, we can see that:

(2c + b) = 10    (because the units digit of 540 is 0)

Then, we can see that:

(b + c + a + 1 ) = 14   (the one comes from the previous 10)

finally:

(2a + b + 1) = 5   (the one comes from the previous 14)

Then we can rewrite:

(2*c + b) = 10

(b + c + a) = 14 -1  = 13

(2a + b) = 5 - 1 = 4

Now we can replace these 3 in the equation:

(b + 2*c)*100 + (c + a + b)*10 + (2a + b) = X

(10)*100 + (13)*10 + 4 = X

1000 + 130 + 4 = X

1134 = X

The sum of the 3 largest numbers is 1134.

Solve the inequality. q/3 < 11

Answers

Answer:

[tex]q < 33[/tex]

Step-by-step explanation:

[tex] \frac{q}{3} < 11[/tex]

[tex]q < 11 \times 3[/tex]

[tex]q < 33[/tex]

Hope it is helpful....

Answer:

33

Step-by-step explanation:

q/3<11

q=11×3

q=33

:. The answer is 33.

Pls answer this pls pls

Answers

1) 520

2)60

3)2600

4)3760

Answer:

1. 520

2. 60

3. 2600

4. 3760

Step-by-step explanation:

[tex]{hope it helps}}[/tex]

Solve. Algebra 1
14=4-6x-2

Answers

Answer:

Step-by-step explanation:

resolver. Álgebra 1

14=4-6x-2

4-6x-2 : 2

Rpta: 2

According to a survey of 1000 families of any town in 2010 A.D.,794 families have radio and 187 families have television.lf 63 families do not have any one of them,then how many families will have both radio and television?Find it.​

Answers

Answer:

the families who use both radio and television is 44.

at a grocery store, an uncooked beef roast is on sale for $5.99/lb. At the same grocery store, prepared roast beef is available at the deli for $2.99/100g. How much more expensive is the deli roast compared to the uncooked roast?

Answers

Answer:

$7.58/lb

Step-by-step explanation:

raw roast: $5.99 per lb

deli roast: $2.99 for 100g

We know the price per lb of the raw roast.

Let's find the price per lb of the deli roast.

1 lb = 454 grams

454 grams / 100 grams = 4.54

1 lb is 4.54 times 100 g

If we multiply the price of 100g of deli roast by 4.54, we get teh price per lb.

$2.99/lb * 4.54 = $13.57/lb

raw roast: $5.99/lb

deli roast: $13.57/lb

difference in price of 1 lb: $13.57 - $5.99 = $7.58

Answer: $7.58/lb

Answer:

.01669 $/g or $7.57 $/lb

Step-by-step explanation:

$2.99/100g = .0299 $/g

1 lb = 453.59237 g.

$5.99/453.59237 g= .0132$/g

Show all work to multiply (3+ Square root of -16)(6- square root of -64)

Answers

Answer:

1042-96i

Step-by-step explanation:

(3 + 16i)(6 - 64i)

6(3 + 16i)(3 - 32i)

(6) 9 - 96i + 48i +1024

A personal narrative essay is usually written in and in .

Answers

Answer:

in the first person participle.

Step-by-step explanation:

Yep yep looks about right

Given a list of candy in a field called Item and prices called Price on an empty PivotTable, how would you find out how many of each type of candy you have?

Answers

Answer:

Drag Item to both the VALUES and the ROWS area

Step-by-step explanation:

To find out how many of each type of candy you have; Drag Item to both the VALUES and the ROWS area

Function A and Function B are linear functions. Function A x y – 10 – 14 – 1 – 5 9 5 Function B y=2x+4 Which statement is true?

Answers

Answer:

See explanation

Step-by-step explanation:

Function A is not clear; I will use the following in place of function A

Function A:

[tex]x \to\ 1 |\ 3 |\ 4 |\ 6[/tex]

[tex]y \to -1|\ 3|\ 5|\ 9[/tex]

Function B:

[tex]y = 2x + 4[/tex]

Required

Compare both functions

For linear functions, we often compare the slope and the y intercepts only.

Calculating the slope of function A, we have:

[tex]m = \frac{y_2 - y_1}{x_2 - x_1}[/tex]

Where:

[tex](x_1,y_1) = (1,-1)[/tex]

[tex](x_2,y_2) = (3,3)[/tex]

So, we have:

[tex]m = \frac{3 - -1}{3 - 1}[/tex]

[tex]m = \frac{4}{2}[/tex]

[tex]m = 2[/tex]

To calculate the y intercept, we set [tex]x = 0[/tex], then solve for y

i.e.[tex](x,y) = (0,y)[/tex]

Using the slope formula, we have:

[tex]m = \frac{y_2 - y_1}{x_2 - x_1}[/tex]

Where:

[tex]m = 2[/tex]

[tex](x_1,y_1) = (0,y)[/tex]

[tex](x_2,y_2) = (3,3)[/tex]

So, we have:

[tex]2 = \frac{3 - y}{3 - 0}[/tex]

[tex]2 = \frac{3 - y}{3}[/tex]

Multiply by 3

[tex]6 = 3 - y[/tex]

Collect like terms

[tex]y = 3 - 6[/tex]

[tex]y = -3[/tex]

So, for function A:

[tex]m = 2[/tex] -- slope

[tex]y = -3[/tex] --- y intercept

For function B

[tex]y = 2x + 4[/tex]

A linear function is represented as:

[tex]y = mx + b[/tex]

By comparison

[tex]m = 2[/tex] --- slope

[tex]b = 4[/tex] --- y intercept

By comparing the results of both functions, we have the following conclusion:

Functions A and B have the same slope (i.e. 2)

Function B has a greater y intercept (i.e. 4)

Evaluate the expression,
32 +6*22-42=23

Answers

Answer:

d

Step-by-step explanation:

Fourth and fifth graders planted trees alongside the road. The distance between two trees is 5 meters how many trees did they plant if the distance between the first and last trees is 200 meters?

Answers

Hey!!

[tex]$\mathcal{PROCESS:}$[/tex]

[tex]{\displaystyle \mathbb {DATA \ \ TO\ \ TAKE \ \ INTO \ \ ACCOUNT}[/tex]

The distance between two trees is 5 meters.the distance between the first and last trees is 200 meters

To know the result of trees we will do the following steps...

We will divide the distance from the first to the last tree by 5, and with this we would have the result:

[tex]200/5=?[/tex]

Result:

40 trees were planted

ATT- Brook2007s

Answer:

41 Trees

Step-by-step explanation:

= (b) Use your calculator to determine the exact value of 35.23 x 3.02 Exactly Correct to 2 decimal place

Answers

Answer:

The exact value is 106.40

Step-by-step explanation:

The exact value of the given operation obtained with the calculator is:

[tex] 35.23 \times 3.02 = 106.3946 [/tex]

The result expressed in 2 decimal places is:

[tex] 106.3946 = 106.40 [/tex]  

Therefore, the exact value is 106.40

I hope it helps you!

A solid wooden block in the shape of a rectangular prism has a length, width and height

Answers

Step-by-step explanation:

Volume of the block

[tex] = lbh[/tex]

[tex] = \frac{3}{8} \times \frac{1}{8} \times \frac{5}{8} [/tex]

[tex] = \frac{3 \times 5}{ {8}^{3} } [/tex]

[tex] = \frac{15}{512} [/tex]

What is the volume of the cylinder to the nearest whole number?

Answers

Answer:

[tex]{ \bf{volume = \pi {r}^{2}h }} \\ = 3.14 \times {7.5}^{2} \times 20 \\ { \tt{volume = 3534.3 \: {cm}^{2} }}[/tex]

Relationships between x and y

Answers

Answer:

so both tables are proportional

1. in the green table

y divided by x =

1/5 for each row

2. in the blue table

y divided by x =

2/5 for each row

Step-by-step explanation:

/ means divided by

divide y by x

for both tables

in each table

if y divided by x

show the same number

for every row

then its proportional

1. in the green table

y divided by x =

1/5 for each row

2. in the blue table

y divided by x =

2/5 for each row

so both tables are proportional

Question 5 can I have as much as you can do please

Answers

Answer:

x = 12

Step-by-step explanation:

The sum of angles in a triangle is 180°

Thus,

(x + 20) + (10x) + (x + 16) = 180°

Collect like terms

x + 10x + x + 20 + 16 = 180°

12x + 36 = 180°

12x = 180 - 36

12x = 144

Divide both sides by 12

x = 144 ÷ 12

x = 12

Substitute the value of x into the question

Therefore,

x + 20 = 12 + 20

= 32°

10x = 10(12)

= 120°

x + 16 = 12 + 16

= 28°

The price of a radio is marked as rs 7500 if shopkeeper allows 20 % Discount and adds 13 % VAT . how much customer pay for radio​

Answers

Answer:

a) A shopkeeper marked the price of an article a certain percentage above the cost price and he allowed 16% discount to make 5% profit. If a customer paid Rs 9.492 with 13% VAT to buy the article, by what percent is the marked price above the cost of the price article? (I hope this helps)

Other Questions
The phase of the business cycle with the highest level of GDP and the lowest unemployment rate is called an? Please help me with this Find the quotient: 63/-9 The table shows information about water used in a household.The value for April is missing.The mean monthly water used for the six months is 18 mWork out the value for April. What is the value of this expression? Find functions f(x) and g(x) so the given function can be expressed as h(x) = f(g(x)). (Use non-identity functions for f(x) and g(x).)h(x) = 5/x-4 Tell whether each probability of the event happening is likely or unlikely to happen. Write L if it is likely to happen and U if unlikely to happen on the space before each number.__ 1. 2:3 __ 2. 4:15 __ 3. 3/10__ 4. 13/21__ 5. 6/16 __ 6. 8:11 __ 7. 9:20__ 8. 11:25__ 9. 5/16__ 10. 7/12 __ 11. 6:13__ 12. 4:9__ 13. 2:5 __ 14. 19/45 __ 15. 12/25Please make it quick A game has a saddle point when the maximin payoff value equals the minimax payoff value. true or false The length of the box is 15 centimeters, the breadth of the box is 20 centimeter, the height of a box, 20 centimeter fine its volume. Step by step What should you substitute for y in the bottom equation to solve the system by the substitution method?A. y=3x+15B. y =-x-5C. y=x+5D. y=-3-15 Pine Company provides for doubtful accounts expense at the rate of 2% of credit sales. The following data are available for 20X4: Allowance for doubtful accounts, 1/1/X4 $18,000 Accounts written off as uncollectible during 20X4 $20,000 Collection of accounts written off in prior years (customer credit was reestablished) $5,000 Credit sales year ended 12/31/X4 $1,500,000 The allowance for doubtful accounts balance at December 31, 20X4, should be which linear inequality represents the graph below?A. y < -1/4x-4B. y < 4x-4C. y < -1/4x+4D. y < -4x+4 AcuveClick to review the online content. Then answer the question(s) below, using complete sentences. Scroll down to view additional questionsOnline Content: Site 1Explain how European Union countries can retain their cultural identities. Include two examples of European products,BIUS X, X"22:ITSave and ExitSubmit The cost depreciation approach of estimating value has two elements, they areSelect one:a. the land value plus the value of the improvements on the landb. the land value and area improvementsc. comparable improvements plus land valued. none of the above ora started watching a movie at 2:45 p.m. She watched the movie for hours before stopping the movie for hours to eat dinner. After dinner, Nora finished watching the remaining hours of the movie. At what time did the movie end? How is the Elizabeth river important to the US development NEED HELP WITH THIS !!!!Any help is great So I'm going into my freshman year in high school on August 12th and I wanted to ask some questions to you guys what can I do to survive my 9th grade year also study tips and things to do to be really good in class supplies I can use and how I can have a good GPA also want to talk about final exams and all that stuff A typical incandescent light bulb consumes 75 W of power and has a mass of 20 g. You want to save electrical energy by dropping the bulb from a height great enough so that the kinetic energy of the bulb when it reaches the floor will be the same as the energy it took to keep the bulb on for 2.0 hours. From what height should you drop the bulb, assuming no air resistance and constant g? Determine the measure of ZA.45.657.755.232.3